2020 AMC 10A 试题/第1题
第1题
满足条件的 \( x \) 的值为多少?
\[ x - \frac{3}{4} = \frac{5}{12} - \frac{1}{3}? \]
(A) \( - \frac{2}{3} \) (B) \( \frac{7}{36} \) (C) \( \frac{7}{12} \) (D) \( \frac{2}{3} \) (E) \( \frac{5}{6} \)
解答
两边同时加上 \( \frac{3}{4} \) ,
\( x = \frac{5}{12} - \frac{1}{3} + \frac{3}{4} = \frac{5}{12} - \frac{4}{12} + \frac{9}{12} = 0\left( \mathbf{E}\right) \frac{5}{6}. \)
2020 AMC 10A 试题/第2题
第2题
数 \( 3,5,7, a, \) 与 \( b \) 的平均值(算术平均数)为15,求 \( a \) 与 \( b \) 的平均值。
(A) 0 (B) 15 (C) 30 (D) 45 (E) 60
解答
数 \( 3,5,7, a, \) 与 \( b \) 的算术平均数等于 \( \frac{3 + 5 + 7 + a + b}{5} = \frac{{15} + a + b}{5} = {15} \) 。解得 \( a + b \) ,从而 \( a + b = {60} \) 。将 \( a \) 与 \( b \) 相加后除以2,得到平均值 \( \frac{60}{2} = \) (C)30
2020 AMC 10A 试题/第3题
第3题
假设 \( a \neq 3 \) 、 \( b \neq 4 \) 和 \( c \neq 5 \) ,则下列表达式的最简值是多少?
\[ \frac{a - 3}{5 - c} \cdot \frac{b - 4}{3 - a} \cdot \frac{c - 5}{4 - b} \]
(A) -1 (B) 1 (C) \( \frac{abc}{60} \) (D) \( \frac{1}{abc} - \frac{1}{60} \) (E) \( \frac{1}{60} - \frac{1}{abc} \)
解答
注意是乘以。同样,是乘以 \( 4 - b \) ,并且是
乘以 \( 5 - c \) 。因此,给定分数的乘积等于
2020 AMC 10A 试题/第4题
题目
一位司机以每小时60英里(miles per hour)的速度行驶2小时,期间她的汽车每加仑(gallon)汽油可行驶30英里(miles per gallon)。她每英里获得 \( \ $ {0.50} \) 美元,唯一支出是每加仑 \( \ $ {2.00} \) 美元的汽油费。扣除这笔费用后,她的净时薪是多少美元/小时?
(A) 20 (B) 22 (C) 24 (D) 25 (E) 26 解答
由于司机每小时行驶60英里,每小时耗油2加仑,因此她每小时在汽油上花费 \( \ $ 4 \) 美元。如果她每英里获得 \( \ $ {0.50} \) 美元,则每小时
驾驶可获得
2020 AMC 10A 试题/第5题
第5题
求所有满足 \( \left| {{x}^{2} - {12x} + {34}}\right| = 2 \) 的实数 \( x \) 之和。
(A) 12 (B) 15 (C) 18 (D) 21 (E) 25 解答1
将方程拆分为两种情况:绝对值内的值为正和非正。
情况1:
方程得到 \( {x}^{2} - {12x} + {34} = 2 \) ,等于
\( \left( {x - 4}\right) \left( {x - 8}\right) = 0 \) 因此,正数情况下的两个值为4和8。
情况2:
类似地,对绝对值符号内的值取非正情况得到 \( - {x}^{2} + {12x} - {34} = 2 \) 。因式分解并化简得 \( {\left( x - 6\right) }^{2} = 0 \) ,因此该情况下的唯一值为6。
将所有值相加得到 \( 4 + 8 + 6 = {(C}){18} \) 。
解法2
我们有方程 \( {x}^{2} - {12x} + {32} = 0 \) 和 \( {x}^{2} - {12x} + {36} = 0 \) 。
注意到第二个方程是完全平方,在 \( x = 6 \) 处有二重根,而第一个方程有实根。根据韦达定理(Vieta’s formulas),第一个方程的根之和为12。
\[ {12} + 6 = \left( \begin{array}{l} \mathrm{C} \end{array}\right) {18} \]
2020 AMC 10A 试题/第6题
题目
有多少个4位正整数(即介于1000到9999之间,含端点)的所有数字均为偶数且能被5整除?
(A) 80 (B) 100 (C) 125 (D) 200 (E) 500
解答
对于所有能被5整除的数,个位必须是0或5。然而,根据题设限制,该位必须为偶数,因此个位只有唯一选择(0)。对于中间两位,可从 \( \left\lbrack {0,8}\right\rbrack \) 中任选一个偶数,即共有5种选择。对于首位,同理可知其不能为0,因此有4种可能。于是,根据乘法原理,我们得到 \( 4 \times 5 \times 5 \times 1 = \left| {\text{(B) }{100}}\right| \) 。~ciceronii
2020 AMC 10A 试题/第7题
以下题目同时出现在2020 AMC 12A第5题和2020 AMC 10A第7题,因此两题均重定向至此页。
题目
从-10到14(含)的25个整数可以排成一个5×5方阵,使得每一行、每一列以及两条主对角线上的数字之和都相同。这个共同的和是多少?
(A) 2 (B) 5 (C) 10 (D) 25 (E) 50
解决方案
不失一般性,考虑该方阵中的五行。每行的数字之和必须相等,这意味着方阵中所有数字之和除以5即为每行的总值。这25个整数的总和为
\( - {10} + 9 + \ldots + {14} = {11} + {12} + {13} + {14} = {50} \) ,以及常见的
总和为 \( \frac{50}{5} = \) (C)10。
解决方案 2
取该集合中间5个值之和(它们最终将成为每一行的平均值)。我们得到 \( 0 + 1 + 2 + 3 + 4 = 0 \) (C) 10作为答案。~Baolan
解决方案 3
取首项-10与末项14的平均值,可得该集合的均值为2。每行、每列或对角线均有5个数,因此公共和为 \( 5 \cdot 2 \) ,即10。~Arctic_Bunny,KINGLOGIC 编辑
2020 AMC 10A 试题/第8题
问题
的值是多少
\( 1 + 2 + 3 - 4 + 5 + 6 + 7 - 8 + \cdots + {197} + {198} + {199} - {200}? \)
(A) 9,800 (B) 9,900 (C) 10,000 (D) 10,100 (E)10,200
解决方案1
将偶数与奇数分开。若将每两个偶数归为一组并相加,总和为 \( {50} \cdot \left( {-2}\right) = - {100} \) 。对奇数求和等同于对前100个奇数求和,结果为 \( {100}^{2} = {10000} \) 。将两者相加,我们得到答案 \( \left( \mathrm{B}\right) {9900} \) 。
解法2(暴力法)
我们可以把整个求和拆成4个整数位,其中总和为 \( {2x} \) ,而 \( x \) 是该位中的第一个整数。我们发现每个序列的第一个和都是 \( {4x} - 3 \) ,将其代入整个序列的50位得到 \( 1 + 2 + 3 + \cdots + {50} = {1275} \) ,于是可将其代入我们上面得到的每个序列方程的第一项 \( 4\left( {1275}\right) - 3\left( {50}\right) = {4950} \) ,因此每一位的和就是 \( {2x} \) ,而我们只算出了 \( x \) 的值,整个序列的和为 \( {4950} \cdot 2 = \left( B\right) {9900} \) -middletonkids
解决方案3
另一种解决方案是将所有内容相加,再减去不需要的部分。第一步是求解
\( 1 + 2 + 3 + 4 + 5 + 6 + 7 + 8 + \cdots + {197} + {198} + {199} + {200} \) 为此,我们只需将200与201相乘再除以2,得到20100。下一步是减去带负号的数。这一步实际上得做两次,因为我们既要剔除本不该加的数,又要从问题中再减去它们。接着可见,从4到200以4为步长递增,共有50个数需要减去。为此,可计算50乘51除以2,再乘以4,因为我们按4计数而非按1计数。所得结果为5100,但记住,这一步需重复两次。完成后,我们得到10200。最后,只需计算 \( {20100} - {10200} \) ,最终答案为 \( |\mathrm{\left( B\right) }{9900} \)
- 菲尼亚斯1500
解决方案 4
在此解法中,我们将每4项归为一组。我们的分组应为: \( 1 + 2 + 3 - 4 = 2 \) 、 \( 5 + 6 + 7 - 8\dot{} = {10},9 + {10} + {11} - {12} = {18},\ldots \)
\( {197} + {198} + {199} - {200} = {394} \) 。我们将它们相加,得到如下表达式: \( 2 + {10} + {18} + \ldots + {394} \) 。这可以改写为
\( 8 * \left( {0 + 1 + 2 + \ldots + {49}}\right) + {100} \) 我们将其加入get(B9900)。~宝蓝
解决方案 5
我们可以把这个长求和拆成每四个整数一组。先计算前几组的和,得到 \( 1 + 2 + 3 - 4 = 2 \) 、 \( 5 + 6 + 7 - 8 = {10} \) 和 \( 9 + {10} + {11} - {12} = {18} \) 。注意到这是一个递增的等差数列,公差为8。我们可以通过求首项与末项的平均值,再乘以项数,来得到该等差数列的和。首项为 \( 1 + 2 + 3 - 4 \) ,即2;末项为 \( {197} + {198} + {199} - {200} \) ,即394;共有 \( {200} \div 4 \) ,即50项。因此,该数列的和为 \( \frac{\left( {{394} + 2}\right) \cdot {50}}{2} \) ,即(B) 9900。~Arctic_Bunny
2020 AMC 10A 试题/第9题
问题
在学校活动中,一个长凳区段(bench section)可以容纳7名成人或11名儿童。当 \( N \) 个长凳区段首尾相连时,若让相同数量的成人和儿童一起就座,恰好占满所有长凳空间。 \( N \) 的最小可能正整数值是多少?
(A) 9 (B) 18 (C) 27 (D) 36 (E) 77
解决方案
7和11的最小公倍数是77。因此,必须有77名成人和77名儿童。长椅的总数为
\( \frac{77}{7} + \frac{77}{11} = {11} + 7 = {13} \)
解决方案 2
这与解法1类似,基本思路相同,但无需计算最小公倍数(LCM)。由于7和11都是质数,它们的最小公倍数必定是其乘积。因此答案为 \( 7 + {11} = \) (B) 18 . PB
2020 AMC 10A 试题/第10题
以下问题同时出现在2020年AMC 12A第7题和2020年AMC 10A第10题,因此两道题均重定向至此。
问题10
七个立方体,其体积分别为1、8、27、64、125、216和343立方单位,垂直堆叠成一座塔,且立方体的体积自下而上递减。除最底部的立方体外,每个立方体的底面都完全位于其下方立方体的顶面上。求这座塔(包括底部)的总表面积,单位为平方单位。
(A) 644 (B) 658 (C) 664 (D) 720 (E) 749
解决方案 1
每个立方体的体积遵循 \( {n}^{3} \) 递增的模式,因为 \( n \) 介于1到7之间。
我们看到总表面积可分为三部分:立方体的侧面、立方体的顶面,以及 \( 7 \times 7 \times 7 \) 立方体的底面(即 \( 7 \times 7 = {49} \) )。侧面积可表示为总和 \( 4\mathop{\sum }\limits_{{n = 0}}^{1}{n}^{2} \) ,得到560。从结构上看,若从顶部观察这座塔,它实际上构成一个 \( 7 \times 7 \) 正方形,面积为49。因此,总表面积为 \( {560} + {49} + {49} = \) (B)658。此外,对于顶面面积,我们也可求和
\( \mathop{\sum }\limits_{{n = 0}}^{6}\left( {{\left( n + 1\right) }^{2} - {n}^{2}}\right) \) ,同样得到49。
~西塞罗尼
解决方案 2
可以迅速看出,这些立方体的边长是从1到7(含)的整数。
首先,我们将计算立方体的总表面积,忽略重叠部分。该值为
\[ 6\left( {{1}^{2} + {2}^{2} + \cdots + {7}^{2}}\right) = 6\mathop{\sum }\limits_{{n = 1}}^{7}{n}^{2} = 6\left( \frac{7\left( {7 + 1}\right) \left( {2 \cdot 7 + 1}\right) }{6}\right) = 7 \cdot 8 \cdot {15} = {840} \]
然后,我们需要减去立方体重叠的部分。在每一对相邻的立方体之间,较小立方体的一个面被完全覆盖,同时较大立方体的一个面也有相同面积被覆盖。
面。因此,立方体重叠部分的总面积等于 \( 2\mathop{\sum }\limits_{{n = 1}}^{6}{n}^{2} = {182} \) 。减去
从总表面积中减去重叠的表面积,我们得到 \( {840} - {182} = 0 \) (B)658。
~绿宝石方块
解法3(比其他方法稍繁琐)
可以看出,使用立方根(cube roots)的立方体边长均为1到7(含)的整数。
只有边长为1和7的立方体在表面积中有5个面,其余立方体均为3个面。此外,由于立方体是堆叠的,我们需要计算每个 \( {n}^{2} \) 与 \( {\left( n - 1\right) }^{2} \) 边长之间的差值,其中 \( n \) 从7到
2.
于是我们得出如下结论:
\( 5\left( {49}\right) + {13} + 4\left( {36}\right) + {11} + 4\left( {25}\right) + 9 + 4\left( {16}\right) + 7 + 4\left( 9\right) + 5 + 4\left( 4\right) + 3 + 5\left( 1\right) \)
然后我们将所有这些相加,得到 \( \left( B\right) {658} \)
2020 AMC 10A 试题/第11题
以下题目同时出现在2020年AMC 12A第8题和2020年AMC 10A第11题,因此两道题均重定向至此页面。
问题11
以下4040个数字的中位数是多少?
\[ 1,2,3,\ldots ,{2020},{1}^{2},{2}^{2},{3}^{2},\ldots ,{2020}^{2} \]
(A) 1974.5 (B) 1975.5 (C) 1976.5 (D) 1977.5 (E) 1978.5 解法1
我们可以看到 \( {44}^{2} \) 小于2020。因此,在2020之后有1976个4040个数。同时,小于或等于2020的数有2064个。由于 \( {44}^{2} \) 等于1936,它与其他平方数一起将我们的中位数位置向上移动44。我们可以得出整个集合的中位数是2020.5,而 \( {2020.5} - {44} \) 给出1976.5。我们的答案是 \( \left| {\mathbf{\left( C\right) }\;{1976.5}}\right| \)
~aryam
解法2
由于我们要找的是一个含4040项的集合的中位数,因此必须求出第2020项与第2021项的平均值。
因为 \( {45}^{2} = {2025} \) 略大于2020,可知44个完全平方数 \( {1}^{2} \) 到 \( {44}^{2} \) 小于2020,其余均大于2020。于是,从数字1到2020共有 \( {2020} + {44} = {2064} \) 项。由于 \( {44}^{2} \) 比 \( {45}^{2} = {2025} \) 小 \( {44} + {45} = {89} \) ,且比2020小84,我们只需从第2064项2020开始向下数84项,再考虑完全平方项即可。第2020项与第2021项距第2064项分别只有44项和43项,因此只需从2020中减去44和43,即得1976与1977。取二者平均,得到
(C) 1976.5。~emerald_block
解法3
我们需要知道第2020项和第2021项,以求中位数。
我们知道 \( {44}^{2} = {1936} \)
因此数字 \( {1}^{2},{2}^{2},...,{44}^{2} \) 位于1到1936之间。
于是44与1936之和为1980,这意味着1936是第1980个数。
同时注意到 \( {45}^{2} = {2025} \) ,它大于2021。
因此第2020项为 \( {1936} + {40} = {1976} \) ,同理第2021项为1977
求这两个数的中位数,得到(C,1976.5)
~toastybaker
2020 AMC 10A 试题/第12题
题目
三角形 \( {AMC} \) 是等腰三角形,且 \( {AM} = {AC} \) 。中线 \( \overline{MV} \) 与 \( \overline{CU} \) 互相垂直,且 \( {MV} = {CU} = {12} \) 。求 \( \bigtriangleup {AMC} \) 的面积?
(A) 48 (B) 72 (C) 96 (D) 144 (E) 192 解法1
由于四边形 \( {UVCM} \) 的对角线互相垂直,其面积等于对角线长度乘积的一半。又注意到 \( \bigtriangleup {AUV} \) 与三角形 \( {AMC} \) 相似,面积比为 \( \frac{1}{4} \) ,因此 \( \left\lbrack {UVCM}\right\rbrack = \frac{3}{4} \cdot \left\lbrack {AMC}\right\rbrack \) 。于是,
\[ \frac{1}{2} \cdot {12} \cdot {12} = \frac{3}{4} \cdot \left\lbrack {AMC}\right\rbrack \]
\[ {72} = \frac{3}{4} \cdot \left\lbrack {AMC}\right\rbrack \]
\[ \left\lbrack {AMC}\right\rbrack = {96} \rightarrow \left( \mathbf{C}\right) . \]
解法2(梯形)我们知道 \( \bigtriangleup {AUV} \sim \bigtriangleup {AMC} \) ,且其边长比为 \( \frac{1}{2} \) ,故面积比为 \( {\left( \frac{1}{2}\right) }^{2} = \frac{1}{4} \) 。
若 \( \bigtriangleup {AUV} \) 是 \( \bigtriangleup {AMC} \) 面积的 \( \frac{1}{4} \) ,则梯形 \( {MUVC} \) 是 \( \bigtriangleup {AMC} \) 面积的 \( \frac{3}{4} \) 。
设 \( \overline{UC} \) 与 \( \overline{MV}P \) 的交点为。令 \( \overline{UP} = x \) 。则 \( \overline{PC} = {12} - x \) 。由于 \( \overline{UC} \bot \overline{MV} \) , \( \overline{UP} \) 与 \( \overline{CP} \) 分别是三角形 \( \bigtriangleup {MUV} \) 和 \( \bigtriangleup {MCV} \) 的高,这两个三角形的底均为12。
\( \bigtriangleup {MUV} = \frac{x \cdot {12}}{2} = {6x} \) 的面积
\( \bigtriangleup {MCV} = \frac{\left( {{12} - x}\right) \cdot {12}}{2} = {72} - {6x} \) 的面积
将两者相加得到梯形 \( {MUVC} \) 的面积,即 \( {6x} + \left( {{72} - {6x}}\right) = {72}. \)
这是三角形面积的 \( \frac{3}{4} \) ,因此三角形面积为 \( \frac{4}{3} \cdot {72} = \) (C) 96 ~quacker88,图由programjames1绘制
解法3(中线)
作中线 \( \overline{AB} \) 。
由于三角形所有中线交于重心,可知 \( \overline{AB} \) 经过点 \( P \) 。又知中线将彼此分成 \( 2 : 1 \) 的比例,由此可得 \( \overline{PC} \) 。 \( \overline{UP} = 2 : 1 \) ,且两段之和为12,故 \( \overline{PC} \) 与 \( \overline{UP} \) 分别为8和4。
最后,由于中线将三角形分成6个面积相等的部分,只需计算 \( \bigtriangleup {PUM} \) 的面积即可。 \( \overline{PC} = \overline{MP} = 8 \) 。
\( \bigtriangleup {PUM} = \frac{4 \cdot 8}{2} = {16} \) 的面积为。乘以6得
\( 6 \cdot {16} = \left( {\mathbf{C}\mathbf{C}}\right) {96} \)
~quacker88
解法4(三角形)
我们知道 \( {AU} = {UM},{AV} = {VC}, \) ,因此 \( {UV} = \frac{1}{2}{MC} \) 。
由于 \( \angle {UPM} = \angle {VPC} = {90} \) ,我们可以看出 \( \bigtriangleup {UPM} \cong \bigtriangleup {VPC} \) 和 \( \bigtriangleup {UVP} \sim \bigtriangleup {MPC} \) ,其边长比为 \( 1 : 2 \) 。
\[ \text{So}{UP} = {VP} = 4\text{,}{MP} = {PC} = 8\text{.} \]
由此可知 \( S{\bigtriangleup {UPM}} = {16} \) ,梯形 \( M{UVC} \) 的面积为72。
如解法1所述, \( S\bigtriangleup {AMC} = {72}/\frac{3}{4} = \) (C)96
-QuadraticFunctions,解法1作者不详
解法5(仅用勾股定理)
设 \( {AB} \) 为高。由于中线互相分成 \( 2 : 1 \) 的比例,且中线长度为12,我们有 \( {PC} = {MP} = 8 \) 和 \( {UP} = {UV} = 4 \) 。从直角三角形 \( \bigtriangleup {MUP} \) ,
\[ M{U}^{2} = M{P}^{2} + U{P}^{2} = {8}^{2} + {4}^{2} = {80}, \]
因此 \( {MU} = \sqrt{80} = 4\sqrt{5} \) 。由于 \( {CU} \) 是中线, \( {AM} = 8\sqrt{5} \) 。从直角三角形 \( \bigtriangleup {MPC} \) ,
\[ M{C}^{2} = M{P}^{2} + P{C}^{2} = {8}^{2} + {8}^{2} = {128}, \]
这意味着 \( {MC} = \sqrt{128} = 8\sqrt{2} \) 。根据对称性
\[ {MB} = \frac{8\sqrt{2}}{2} = 4\sqrt{2}. \]
对直角三角形 \( \bigtriangleup {MAB} \) 应用勾股定理,得到
\[ A{B}^{2} = A{M}^{2} - M{B}^{2} = 8{\sqrt{5}}^{2} - 4{\sqrt{2}}^{2} = {288}\text{, so} \]
\( {AB} = \sqrt{288} = {12}\sqrt{2} \) 。那么 \( \bigtriangleup {AMC} \) 的面积为
解法6(作图法)
(不推荐)将给出的图形(恰好是按比例绘制的)转绘到方格纸上。由于选项差距较大,数格子应能得到可靠结果。-Lingjun
解法7
给定一个三角形,其两条中线互相垂直,长度分别为 \( x \) 和 \( y \) ,则面积为 \( \frac{2xy}{3} = \) (C)96
解法8(最快)
连接线段 \( {UV} \) ,很容易看出四边形 \( {UVMC} \) 的面积为对角线乘积的一半,即72。现在,求解三角形 \( A{UV} \) 的面积是一个选择,但图示表明 \( A{UV} \) 的面积小于四边形,这意味着 \( {AMC} \) 的面积小于 \( {72} * 2 \) 但大于72,因此唯一可能的选项是 \( \left( {\mathrm{C}\text{)}{96}}\right) \) 。
2020 AMC 10A 试题/第13题
问题13
一只青蛙从点(1,2)出发,开始一系列跳跃,每次跳跃平行于坐标轴之一,长度为1,且每次跳跃的方向(上、下、右或左)独立随机选择。当青蛙到达以 \( \left( {0,0}\right) ,\left( {0,4}\right) ,\left( {4,4}\right) , \) 和(4,0)为顶点的正方形的一条边时,序列结束。求跳跃序列在正方形的垂直边上结束的概率。
(A) \( \frac{1}{2} \) (B) \( \frac{5}{8} \) (C) \( \frac{2}{3} \) (D) \( \frac{3}{4} \) (E) \( \frac{7}{8} \)
解决方案
画出正方形后很容易看出,如果青蛙向左跳,它会立刻碰到正方形的竖直边界,因此这种情况发生的概率是 \( \frac{1}{4} * 1 = \frac{1}{4} \) 。如果青蛙向右跳,它会到达正方形中心(2,2);由于对称性(青蛙到正方形各边的距离相等),它碰到正方形竖直边的概率是 \( \frac{1}{2} \) ,这种情况发生的概率为 \( \frac{1}{4} * \frac{1}{2} = \frac{1}{8} \) 。
如果青蛙向上或向下移动,它将撞向离它最近和最远的角落所形成的对称线,并且它到两条较近边的距离相等,到两条较远边的距离也相等。它撞向垂直墙的概率为 \( \frac{1}{2} \) 。由于青蛙向上或向下移动的概率为 \( \frac{1}{2} \) ,因此该情况的总概率为 \( \frac{1}{2} * \frac{1}{2} = \frac{1}{4} \) ,将所有情况相加,
\[ \frac{1}{4} + \frac{1}{8} + \frac{1}{4} = \frac{5}{8} \Rightarrow \text{ (B) }\frac{5}{8}. \]
解决方案 2
假设我们有一个4×4的网格,我们用分类讨论的方法来解决这个问题。A是青蛙的初始位置,B和C是第二次跳跃可能到达的位置,O代表其他所有格子。如果我们落在C,就表示到达了竖直边;如果落在B,由于我们处于对称位置,到达水平边和竖直边的概率相等。因此,落在C的概率是 \( 1/4 \) ,落在B的概率是 \( 3/4 \) 。由于C代表“成功”,而B意味着有一半的成功机会,我们计算 \( 1 \cdot C + \frac{1}{2} \cdot B \) 。
\[ 1 \cdot \frac{1}{4} + \frac{1}{2} \cdot \frac{3}{4} \]
\[ \frac{1}{4} + \frac{3}{8} \]
我们得到 \( \frac{5}{8} \) ,或者 \( B \)
\[ \text{O O O O O} \]
O B O O O
\[ \text{C A B O O} \]
O B O O O O O O O O
-是的
解决方案 3
如果青蛙位于两条对角线之一,落在垂直或水平方向的概率均变为 \( \frac{1}{2} \) 。由于它从(1,2)出发,第一次跳跃有 \( \frac{3}{4} \) 的概率(向上、向下或向右)到达对角线,有 \( \frac{1}{4} \) 的概率(向左)到达垂直边。落在垂直方向的概率为 \( \frac{1}{4} + \frac{3}{4} * \frac{1}{2} = 0 \) (B) \( \frac{5}{8} \) 。Lingjun
解法4(完整状态)
令 \( {P}_{\left( x, y\right) } \) 表示青蛙位于(x, y)时,其跳跃序列以碰到竖直边结束的概率。注意,根据关于直线 \( x = 2 \) 的反射对称性, \( {P}_{\left( 1,2\right) } = {P}_{\left( 3,2\right) } \) 。类似地, \( {P}_{\left( 1,1\right) } = {P}_{\left( 1,3\right) } = {P}_{\left( 3,1\right) } = {P}_{\left( 3,3\right) } \) ,且 \( {P}_{\left( 2,1\right) } = {P}_{\left( 2,3\right) } \) 。现在,我们通过考虑从该点向上、下、左、右移动的概率,为这些点/状态的概率建立方程:
\[ {P}_{\left( 1,2\right) } = \frac{1}{4} + \frac{1}{2}{P}_{\left( 1,1\right) } + \frac{1}{4}{P}_{\left( 2,2\right) } \]
\[ {P}_{\left( 2,2\right) } = \frac{1}{2}{P}_{\left( 1,2\right) } + \frac{1}{2}{P}_{\left( 2,1\right) } \]
\[ {P}_{\left( 1,1\right) } = \frac{1}{4} + \frac{1}{4}{P}_{\left( 1,2\right) } + \frac{1}{4}{P}_{\left( 2,1\right) } \]
\[ {P}_{\left( 2,1\right) } = \frac{1}{2}{P}_{\left( 1,1\right) } + \frac{1}{4}{P}_{\left( 2,2\right) } \]
我们得到一个含4个变量的4方程组,因此可以解出这些概率。将第二个方程代入第四个方程得
\[ {P}_{\left( 2,1\right) } = \frac{1}{2}{P}_{\left( 1,1\right) } + \frac{1}{4}\left( {\frac{1}{2}{P}_{\left( 1,2\right) } + \frac{1}{2}{P}_{\left( 2,1\right) }}\right) \]
\[ {P}_{\left( 2,1\right) } = \frac{8}{7}\left( {\frac{1}{2}{P}_{\left( 1,1\right) } + \frac{1}{8}{P}_{\left( 1,2\right) }}\right) = \frac{4}{7}{P}_{\left( 1,1\right) } + \frac{1}{7}{P}_{\left( 1,2\right) } \]
再将第三个方程代入上式得
\[ {P}_{\left( 2,1\right) } = \frac{4}{7}\left( {\frac{1}{4} + \frac{1}{4}{P}_{\left( 1,2\right) } + \frac{1}{4}{P}_{\left( 2,1\right) }}\right) + \frac{1}{7}{P}_{\left( 1,2\right) } \]
\[ {P}_{\left( 2,1\right) } = \frac{7}{6}\left( {\frac{1}{7} + \frac{2}{7}{P}_{\left( 1,2\right) }}\right) = \frac{1}{6} + \frac{1}{3}{P}_{\left( 1,2\right) }\left( *\right) \]
(*)
接着,将第二和第三个方程代入第一个方程得
\[ {P}_{\left( 1,2\right) } = \frac{1}{4} + \frac{1}{2}\left( {\frac{1}{4} + \frac{1}{4}{P}_{\left( 1,2\right) } + \frac{1}{4}{P}_{\left( 2,1\right) }}\right) + \frac{1}{4}\left( {\frac{1}{2}{P}_{\left( 1,2\right) } + \frac{1}{2}{P}_{\left( 2,1\right) }}\right) \]
\[ {P}_{\left( 1,2\right) } = \frac{3}{8} + \frac{1}{4}{P}_{\left( 1,2\right) } + \frac{1}{4}{P}_{\left( 2,1\right) } \]
现在将(*)代入上式,我们得到
\[ {P}_{\left( 1,2\right) } = \frac{3}{8} + \frac{1}{4}{P}_{\left( 1,2\right) } + \frac{1}{4}\left( {\frac{1}{6} + \frac{1}{3}{P}_{\left( 1,2\right) }}\right) \]
\[ {P}_{\left( 1,2\right) } = \frac{3}{2} \cdot \frac{5}{12} = 6\mathrm{\;B}2\frac{5}{8} \]
2020 AMC 10A 第14题
实数 \( x \) 和 \( y \) 满足 \( x + y = 4 \) 和 \( x \cdot y = - 2 \) 。求
\[ x + \frac{{x}^{3}}{{y}^{2}} + \frac{{y}^{3}}{{x}^{2}} + y? \]
(A) 360 (B) 400 (D) 440 (E) 480
- 注:所有解法均涉及因式分解
解法1
\[ x + \frac{{x}^{3}}{{y}^{2}} + \frac{{y}^{3}}{{x}^{2}} + y = x + \frac{{x}^{3}}{{y}^{2}} + y + \frac{{y}^{3}}{{x}^{2}} = \frac{{x}^{3}}{{x}^{2}} + \frac{{y}^{3}}{{x}^{2}} + \frac{{y}^{3}}{{y}^{2}} + \frac{{x}^{3}}{{y}^{2}} \]
继续合并
\[ \frac{{x}^{3} + {y}^{3}}{{x}^{2}} + \frac{{x}^{3} + {y}^{3}}{{y}^{2}} = \frac{\left( {{x}^{2} + {y}^{2}}\right) \left( {{x}^{3} + {y}^{3}}\right) }{{x}^{2}{y}^{2}} = \frac{\left( {{x}^{2} + {y}^{2}}\right) \left( {x + y}\right) \left( {{x}^{2} - {xy} + {y}^{2}}\right) }{{x}^{2}{y}^{2}} \]
由已知可得 \( {x}^{2}{y}^{2} = 4 \) 。同时,
\[ {\left( x + y\right) }^{2} = {x}^{2} + {2xy} + {y}^{2} = {16} \]
这意味着 \( {x}^{2} + {y}^{2} = {20} \) 。将此信息代入
\( \frac{\left( {{x}^{2} + {y}^{2}}\right) \left( {x + y}\right) \left( {{x}^{2} - {xy} + {y}^{2}}\right) }{{x}^{2}{y}^{2}} \) ,我们有
\( \frac{\left( {20}\right) \left( 4\right) \left( {22}\right) }{4} = {20} \cdot {22} = {(D)440}. \sim \) PCChess
解法2
如上所述,我们需要计算 \( \frac{\left( {{x}^{2} + {y}^{2}}\right) \left( {{x}^{3} + {y}^{3}}\right) }{{x}^{2}{y}^{2}} \) 。注意 \( x, y \) 是 \( {x}^{2} - {4x} - 2 \) 的根,因此 \( {x}^{3} = 4{x}^{2} + {2x} \) 且 \( {y}^{3} = 4{y}^{2} + {2y} \) 。于是 \( {x}^{3} + {y}^{3} = 4\left( {{x}^{2} + {y}^{2}}\right) + 2\left( {x + y}\right) = 4\left( {20}\right) + 2\left( 4\right) = {88} \) ,其中 \( {x}^{2} + {y}^{2} = {20} \) 和 \( {x}^{2}{y}^{2} = 4 \) 与前一解法相同。因此答案为 \( \frac{\left( {20}\right) \left( {88}\right) }{4} = \) (D)440。
Emathmaster
解法3
注意 \( \left( {{x}^{3} + {y}^{3}}\right) \left( {\frac{1}{{y}^{2}} + \frac{1}{{x}^{2}}}\right) = x + \frac{{x}^{3}}{{y}^{2}} + \frac{{y}^{3}}{{x}^{2}} + y \) 。现在,我们只需找出 \( {x}^{3} + {y}^{3} \) 和 \( \frac{1}{{y}^{2}} + \frac{1}{{x}^{2}} \) 的值。
回忆 \( {x}^{3} + {y}^{3} = \left( {x + y}\right) \left( {{x}^{2} - {xy} + {y}^{2}}\right) , \) 且 \( {x}^{2} - {xy} + {y}^{2} = {\left( x + y\right) }^{2} - {3xy}. \) 。我们可以解第二个方程,解得 \( {4}^{2} - 3\left( {-2}\right) = {22}. \) 。将其代入第一个方程,得到 \( {x}^{3} + {y}^{3} = 4\left( {22}\right) = {88} \) 。
为了求 \( \frac{1}{{y}^{2}} + \frac{1}{{x}^{2}} \) 的值,我们通分以便相加,得到 \( \frac{{x}^{2}}{{x}^{2}{y}^{2}} + \frac{{y}^{2}}{{x}^{2}{y}^{2}} = \frac{{x}^{2} + {y}^{2}}{{\left( xy\right) }^{2}} \) 。回忆 \( {x}^{2} + {y}^{2} = {\left( x + y\right) }^{2} - {2xy} \) 并解此方程,得 \( {4}^{2} - 2\left( {-2}\right) = {20}. \) 。将其代入第一个方程,得到
\[ \frac{1}{{y}^{2}} + \frac{1}{{x}^{2}} = \frac{20}{{\left( -2\right) }^{2}} = 5. \]
解原方程,得
\( x + \frac{{x}^{3}}{{y}^{2}} + \frac{{y}^{3}}{{x}^{2}} + y = \left( {88}\right) \left( 5\right) = \) (D) 440.
解法4
这基本上是用韦达公式(Vieta’s formulas)硬算 \( x \) 和 \( y \) (我极不推荐,只是写着玩)。
我们利用韦达定理(Vieta’s)建立与 \( x \) 和 \( y \) 相关的二次方程。令 \( x \) 和 \( y \) 为二次方程 \( Q\left( n\right) = {n}^{2} - {4n} - 2 \) 的根(因为 \( x + y = 4 \) ,且 \( {xy} = - 2 \) )。解该二次方程可得根 \( 2 + \sqrt{6} \) 和 \( 2 - \sqrt{6} \) 。 \( x \) 与 \( y \) “可互换”,即无论 \( x \) 或 \( y \) 取哪个解,代入后结果相同。于是我们将 \( 2 + \sqrt{6} \) 代入 \( x \) , \( 2 - \sqrt{6} \) 代入 \( y \) ,得到答案 \( \left| {\left( \mathrm{D}\right) {440}}\right| \) 。
~Baolan
视频解析
https://youtu.be/ZGwAasE32Y4
~lceMatrix 另见
2020 AMC 10A 第15题
题目
从 \( {12}! \) 的正整数因子中随机选取一个。选出的因子是完全平方数的概率可表示为 \( \frac{m}{n} \) ,其中 \( m \) 与 \( n \) 为互质的正整数。 \( m + n \) 是多少?
(A) 3 (B) 5 (C) 12 (D) 18 (E) 23 解答
\( {12}! \) 的质因数分解为 \( {2}^{10} \cdot {3}^{5} \cdot {5}^{2} \cdot 7 \cdot {11} \) ,因此共有 \( {11} \cdot 6 \cdot 3 \cdot 2 \cdot 2 \) 个因子。要得到完全平方因子,每个质因数的指数必须为偶数。注意7和11不能出现在完全平方因子的质因数分解中,因为 \( {12}!. \) 中各仅出现一次。于是共有 \( 6 \cdot 3 \cdot 2 \) 个完全平方数。(对于2,指数可取0、2、4、6、8或10,等等。)选出的因子为完全平方数的概率为~mshell214,由Rzhpamath编辑
视频解析
https://youtu.be/ZGwAasE32Y4
~lceMatrix 另见
2020 AMC 10A 第16题
以下题目同时出现在2020 AMC 12A第16题和2020 AMC 10A第16题,因此两题均重定向至此页面。
题目
在坐标平面上,顶点为 \( \left( {0,0}\right) ,\left( {{2020},0}\right) ,\left( {{2020},{2020}}\right) , \) 和(0,2020)的正方形内随机取一点。该点与某个格点距离不超过 \( d \) 的概率为 \( \frac{1}{2} \) 。(若 \( x \) 与 \( y \) 均为整数,则点(x, y)为格点。) \( d \) 四舍五入到小数点后一位是多少?
(A)0.3(B)0.4(C)0.5(D)0.6(E)0.7
解法1
图示
由Shurong.ge使用Asymptote绘制的图示
注:该图示表示给定 \( {2020} * {2020} \) 正方形的每个单位方格。
解法
我们考虑一个1×1的独立方块。
若从每个角(格点所在位置)画一个半径为 \( d \) 的四分之一圆,则这些圆覆盖的面积应为0.5。因此,考虑到共有四个圆,我们列出
\[ 4 * \frac{1}{4} * \pi {d}^{2} = \frac{1}{2} \]
解 \( d \) ,得 \( d = \frac{1}{\sqrt{2\pi }} \) ,其中当 \( \pi \approx 3 \) 时,我们得到 \( d = \frac{1}{\sqrt{6}} \) ,由此化简可见 \( d \approx {0.4} \Rightarrow d \) (B)0.4。 \( \sim \) Crypthes
\( \mathbf{{Note} : } \) 为更严谨,注意 \( d < {0.5} \) ,因为若 \( d \geq {0.5} \) ,则概率显然大于 \( \frac{1}{2} \) 。这确保上述解法成立,因为若 \( d \geq {0.5} \) ,四分之一圆会重叠。—— \( \mathbf{{Emathmaster}} \)
解法2
与前一解法相同,我们得到方程 \( 4 * \frac{1}{4} * \pi {d}^{2} = \frac{1}{2} \) ,化简为 \( \pi {d}^{2} = \frac{1}{2} = {0.5} \) 。由于 \( \pi \) 略大于3, \( {d}^{2} \) 略小于 \( \frac{0.5}{3} = {0.1}\overline{6} \) 。我们注意到 \( {0.1}\overline{6} \) 略大于 \( {0.4}^{2} = {0.16} \) ,因此 \( d \) 约为 \( \left( {B){0.4}}\right) \) 。——emerald_block
解法3(估算)
如上,我们需估算 \( d = \frac{1}{\sqrt{2\pi }} \) 。
注意我们可近似 \( {2\pi } \approx {6.28} \approx {6.25} \) ,于是 \( \frac{1}{\sqrt{2\pi }} \)
\( \approx \frac{1}{\sqrt{6.25}} = \frac{1}{2.5} = {0.4}. \)
因此答案为 \( \begin{array}{l} \text{ (B) }{0.4} \end{array} \)
~银龙
2020 AMC 10A 试题/第17题
定义
\[ P\left( x\right) = \left( {x - {1}^{2}}\right) \left( {x - {2}^{2}}\right) \cdots \left( {x - {100}^{2}}\right) . \]
有多少个整数 \( n \) 满足 \( P\left( n\right) \leq 0 \) ?
(A) 4900 (B) 4950 (C) 5000 (D) 5050 (E) 5100
解决方案 1
注意 \( P\left( x\right) \) 是许多整数的乘积。我们需要其中一个因子为0,或者有奇数个负因子。
案例1:存在100个整数 \( n \) ,使得 \( P\left( x\right) = 0 \)
情况2:要使负因数的个数为奇数, \( n \) 必须介于一个奇数的平方与一个偶数的平方之间。这意味着 \( n \) 共有 \( 2 + 6 + \cdots + {198} \) 个可能的取值。化简后,共有5000个可能的数。
综上所述, \( n \) 共有(E,5100)种可能的取值。PCChess
解决方案 2
注意到当 \( x \) 介于 \( {100}^{2} \) 与 \( {99}^{2} \) 、 \( {98}^{2} \) 与 \( {97}^{2}\ldots \) 、 \( {2}^{2} \) 与 \( {1}^{2} \) 之间(含端点)时, \( P\left( x\right) \) 为非正,这意味着数值的数量等于
\( \left( {\left( {{100} + {99}}\right) \left( {{100} - {99}}\right) + 1}\right) + \left( {\left( {{98} + {97}}\right) \left( {{98} - {97}}\right) + 1}\right) + \ldots + \left( {\left( {2 + 1}\right) \left( {2 - 1}\right) + 1}\right) \)
这简化为
~泽里克
解法3(末端行为)
我们知道 \( P\left( x\right) \) 是一个100次函数(100-degree function),且其首项系数为正。也就是说, \( P\left( x\right) = {x}^{100} + a{x}^{99} + b{x}^{98} + \ldots + \) (常数)。由于 \( P\left( x\right) \) 的次数为偶数,其两端的行为一致;又因为首项系数为正,所以当 \( x \) 向任意方向趋近时,两端都趋近于 \( \infty \) 。
\[ \mathop{\lim }\limits_{{x \rightarrow - \infty }}P\left( x\right) = \mathop{\lim }\limits_{{x \rightarrow \infty }}P\left( x\right) = \infty \]
因此, \( P\left( x\right) \) 第一次为负是在它与 \( x \) 轴相交于一个 \( x \) 截距并向下穿过时。这发生在 \( {1}^{2} \) ,即最小的截距处。
然而,当它到达下一个截距时,又会回升到正值区域,我们知道这发生在 \( {2}^{2} \) 。而当它到达 \( {3}^{2} \) 时,又会再次跌入负值区域。显然,这一过程将沿着截距不断“蛇形”前进,直到 \( {100}^{2} \) 。
要计算位于 \( x \) 轴下方或轴上的整数个数,我们只需数整数即可。例如,对于之前得到的 \( \left\lbrack {{1}^{2},{2}^{2}}\right\rbrack \) 区间内的整数个数,我们减一、加一。 \( \left( {{2}^{2} - {1}^{2} + 1}\right) = 4 \) 个整数,因此该区间内有四个整数使结果为负。
对所有其他区间重复此操作,我们得到
\( \left( {{2}^{2} - {1}^{2} + 1}\right) + \left( {{4}^{2} - {3}^{2} + 1}\right) + \ldots + \left( {{100}^{2} - {99}^{2} + 1}\right) \) 。继续采用解法2。~quacker88
2020 AMC 10A 试题/第18题
题目
设(a, b, c, d)为一个有序四元组,其中元素不必互异,每个元素取自集合{0,1,2,3}。有多少个这样的四元组满足 \( a \cdot d - b \cdot c \) 为奇数?(例如,(0,3,1,1)就是一个这样的四元组,因为 \( 0 \cdot 1 - 3 \cdot 1 = - 3 \) 为奇数。)
(A) 48 (B) 64 (C) 96 (D) 128 (E) 192
解答
解法1(奇偶性)
为了使 \( a \cdot d - b \cdot c \) 为奇数,考虑奇偶性。我们必须有(偶数)-(奇数)或(奇数)-(偶数)。有 \( 2 \cdot 4 + 2 \cdot 2 = {12} \) 种方式选取数字以得到偶数积。有 \( 2 \cdot 2 = 4 \) 种方式得到奇数积。因此,
使 \( a \cdot d - b \cdot c \) 为奇数的总方式数为 \( 2 \cdot \left( {{12} \cdot 4}\right) = \left( \mathbf{C}\right) {96} \)
-Midnight
解法2(本质上是解法1,但更详细)
考虑奇偶性。我们需要恰好一项为奇数,一项为偶数。由于对称性,可设 \( {ad} \) 为奇数, \( {bc} \) 为偶数,然后乘以2。若 \( {ad} \) 为奇数,则 \( a \) 和 \( d \) 必须均为奇数,因此 \( {ad}. \) 有 \( 2 \cdot 2 = 4 \) 种可能。考虑 \( {bc}. \) ,设 \( b \) 为偶数,则 \( {bc}. \) 有 \( 2 \cdot 4 = 8 \) 种可能。然而, \( b \) 也可为奇数,此时 \( {bc}. \) 又有 \( 2 \cdot 2 = 4 \) 种可能。因此,选择 \( {bc} \) 有12种方式,选择 \( {ad}. \) 有4种方式。于是,考虑对称性后, \( {ad} - {bc}.\left( C\right) \) 共有 \( 2 * 4 * {12} = {96} \) 个取值。
解法3(补集计数)
独立选择任意数字有4种方式,独立选择任意奇数有2种方式。为了得到偶数积,我们计数:
\( \mathrm{P} \) (任意数字) \( \cdot \mathrm{P} \) (任意数字) \( - \mathrm{P}\left( \mathrm{{odd}}\right) \cdot \mathrm{P}\left( \mathrm{{odd}}\right) \) ,即 \( 4 \cdot 4 - 2 \cdot 2 = {12} \) 。得到奇数积的方式可以这样计数: \( \mathrm{P}\left( \mathrm{{odd}}\right) \cdot \mathrm{P}\left( \mathrm{{odd}}\right) \) ,即 \( 2 \cdot 2 \) ,或4。因此,一个积为奇数另一个为偶数: \( 2 \cdot 4 \cdot {12} = O\left( C\right) {96} \) (顺序重要)。 \( \sim \) Anonymous 和 Arctic_Bunny
解法4(解法3的深入版)
我们使用补集计数:若差为偶数,则可减去这些情况。总共有 \( {4}^{4} = {256} \) 种情况。
对于偶数差,我们有(偶)-(偶)或(奇-奇)。
根据解法3:
“独立选择任意数字有4种方式,独立选择任意奇数有2种方式。偶数积:(数字)×(数字)-(奇数)×(奇数): \( 4 \cdot 4 - 2 \cdot 2 = {12} \) 。奇数积:(奇数)×(奇数): \( 2 \cdot 2 = 4 \) 。”
由此,我们轻松算出 \( {256} - {12}^{2} - {4}^{2} = \mathbf{\left( C\right) {96}} \) 。~kevinmathz
在计算偶数积和奇数积的数量时,由于唯一得到奇数积的方式是将两个奇整数相乘,且有2个奇整数,可迅速推出奇数积有 \( 2 \cdot 2 = 4 \) 种可能。
由于积只能是奇数或偶数,且选择积的因数有 \( 4 \cdot 4 = {16} \) 种方式,因此偶数积有 \( {16} - 4 = {12} \) 种可能。~emerald_block
2020 AMC 10A 问题/第19题
题目
如下图所示,一个正十二面体(由12个全等的正五边形面组成的多面体)在太空中漂浮,有两个水平面。注意,与顶面相邻有一圈五个倾斜面,与底面相邻也有一圈五个倾斜面。有多少种方式可以从顶面出发,通过一系列相邻面移动到顶面,使得每个面最多被访问一次,且不允许从底环移动到顶环?
(A) 125 (B) 250 (C) 405 (D) 640 (E) 810
图示
解法
由于我们从顶面出发,最终到达底面,且不能从下环回到上环,也不能重复访问任何面,因此整个路径必须依次经过:顶面、上环的一系列面、下环的一系列面,最后到底面。
在上环,我们首先要选择第一个访问的面,有5种选择。接着,我们决定在上环绕行多远再向下:顺时针绕行1、2、3或4个面,逆时针绕行1、2、3或4个面,或者立即下到下环而不在上环继续访问其他面,共9种选择。
然后,我们需选择下环的第一个面,有2种选择(因为每个上环面恰好与2个下环面相邻);之后在下环的绕行方式同样有9种选择。最后一步即进入底面,完成整个行程。
把所有可选数目相乘,得到 \( 5 \cdot 9 \cdot 2 \cdot 9 = 1 \) (E)810
解法2
将面与顶点互换,即把面当作顶点、顶点当作面。于是,此题便与2016 AIME I第3题(https://artofproblemsolving.com/wiki/index.php/2016_AIME_I_Problems/Problem_3)相同,其答案为 \( \left( \mathbf{E}\right) {810} \) 。——Emathmaster
2020 AMC 10A 试题/第20题
以下题目同时出现在2020年AMC 12A第18题和2020年AMC 10A第20题,因此两道题均重定向至此页面。
问题
四边形 \( {ABCD} \) 满足 \( \angle {ABC} = \angle {ACD} = {90}^{ \circ },{AC} = {20}, \) 和 \( {CD} = {30} \) 。对角线 \( \overline{AC} \) 与 \( \overline{BD} \) 交于点 \( E, \) 且 \( {AE} = 5. \) 。求四边形 \( {ABCD} \) 的面积。
(A) 330 (B) 340 (C) 350 (D) 360 (E) 370
解法1(只需作一条高线)
这一步画好示意图至关重要。由于 \( {AC} = {20} \) 且 \( {CD} = {30} \) ,我们得到 \( \left\lbrack {ACD}\right\rbrack = {300} \) 。现在需要求出 \( \left\lbrack {ABC}\right\rbrack \) ,才能算出整个四边形的面积。从 \( B \) 向 \( {AC} \) 作垂线,垂足记为 \( F \) 。设 \( {FE} = x \) 。因为 \( {AE} = 5 \) ,所以 \( {AF} = 5 - x \) 。通过作这条垂线,我们还能发现两个相似三角形: \( {BFE} \) 和 \( {DCE} \) 。由于 \( {EC} \) 是
\( {20} - 5 = {15} \) ,且 \( {DC} = {30} \) ,我们得到 \( {BF} = {2x} \) 。现在,如果我们重新绘制仅包含 \( {ABC} \) 的图,我们得到 \( {\left( 2x\right) }^{2} = \left( {5 - x}\right) \left( {{15} + x}\right) \) 。接下来,展开、化简并除以最大公因数(GCF),我们得到 \( {x}^{2} + {2x} - {15} = 0 \) 。这可以因式分解为 \( \left( {x + 5}\right) \left( {x - 3}\right) \) 。由于长度不能为负, \( x = 3 \) 。由于 \( x = 3 \) , \( \left\lbrack {ABC}\right\rbrack = {60} \) 。所以
\[ \left\lbrack {ABCD}\right\rbrack = \left\lbrack {ACD}\right\rbrack + \left\lbrack {ABC}\right\rbrack = {300} + {60} = {600} \]
(如果你是视觉学习者,我很抱歉,但现在你有了ciceronii提供的示意图)
~ 奥特曼的解法
\( \sim \) 图表由ciceronii绘制
解法2(专业猜题策略)
我们知道大三角形的面积为300。利用选项可知小三角形的面积是10的倍数,因此两直角边的乘积为
20的倍数。假设两条直角边分别为 \( \sqrt{20a} \) 和 \( \sqrt{20b} \) ,由于斜边为20,我们得到 \( a + b = {20} \) 。尝试较小的整数,当 \( a = 2 \) 和 \( b = {18} \) 时, \( {ab} \) 确实是一个完全平方数。因此三角形的面积为60,所以答案是(D) 360
~tigershark22~(由HappyHuman编辑)
解法3(坐标法)
设这些点依次为 \( A\left( {-{10},0}\right) \) 、 \( B\left( {x, y}\right) \) 、 \( C\left( {{10},0}\right) \) 、 \( D\left( {{10},{30}}\right) \) 和 \( E\left( {-5,0}\right) \) 。由于 \( B \) 位于直线 \( {DE} \) 上,可知 \( y = {2x} + {10} \) 。此外,由于 \( \angle {ABC} = {90}^{ \circ } \) , \( B \) 位于以 \( {AC} \) 为直径的圆上,因此 \( {x}^{2} + {y}^{2} = {100} \) 。联立这些方程解 \( x \) 和 \( y \) ,得到两组解(0,10)和(-8,-6)。我们立即舍去(0,10)这一组,因为 \( y \) 应为
负数。于是,我们得出
解法4(三角法)
设 \( \angle C = \angle {ACB} \) 和 \( \angle B = \angle {CBE}. \) ,在 \( \bigtriangleup {BCE} \) 上应用正弦定理,得到
\[ \frac{BE}{\sin C} = \frac{CE}{\sin B} = \frac{15}{\sin B} \]
再在 \( \bigtriangleup {ABE} \) 上应用正弦定理,得到
\[ \frac{BE}{\sin \left( {{90} - C}\right) } = \frac{5}{\sin \left( {{90} - B}\right) } = \frac{BE}{\cos C} = \frac{5}{\cos B}. \]
两式相除得 \( \tan B = 3\tan C. \) 。现在,
\[ \tan \angle {CED} = 2 = \tan \angle B + \angle C = \frac{4\tan C}{1 - 3{\tan }^{2}C} \]
解该二次方程,取正根(C为锐角),得 \( \tan C = \frac{1}{3} \) 。因此,若 \( {AB} = a, \) ,则 \( {BC} = {3a} \) 且 \( \left\lbrack {ABC}\right\rbrack = \frac{3{a}^{2}}{2}. \) 。由勾股定理, \( {10}{a}^{2} = {400} \Leftrightarrow \frac{3{a}^{2}}{2} = {60} \) ,答案为 \( {300} + {60} \Leftrightarrow \) (D)
(此解法不完整,请哪位补充——Lingjun)已由kc5170完成LaTeX编辑
2020 AMC 10A 试题/第21题
题目
存在唯一一个严格递增的非负整数序列 \( {a}_{1} < {a}_{2} < \ldots < {a}_{k} \) ,使得
\[ \frac{{2}^{289} + 1}{{2}^{17} + 1} = {2}^{{a}_{1}} + {2}^{{a}_{2}} + \ldots + {2}^{{a}_{k}}. \]
求 \( k \) 的值?
(A) 117 (B) 136 (C) 137 (D) 273 (E) 306
解法1
首先,将 \( {2}^{17} \) 替换为 \( a \) 。于是原方程变为 \( \frac{{a}^{17} + 1}{a + 1} = {a}^{16} - {a}^{15} + {a}^{14}\ldots - {a}^{1} + {a}^{0} \) 。现在仅考虑 \( {a}^{16} - {a}^{15} \) ,它等于 \( {a}^{15}\left( {a - 1}\right) = {a}^{15} * \left( {{2}^{17} - 1}\right) \) 。注意到 \( {2}^{17} - 1 \) 等于 \( {2}^{16} + {2}^{15} + \ldots + 1 \) ,因为几何级数求和公式为 \( \frac{{a}^{n} - 1}{a - 1} \) 。因此,可以看出 \( {a}^{16} - {a}^{15} \) 是17个不同2的幂次之和。对 \( {a}^{14} - {a}^{13},{a}^{12} - {a}^{11},\ldots ,{a}^{2} - {a}^{1} \) 的每一对应用同样的方法,可知每一对都是17个不同2的幂次之和。于是得到 \( {17} * 8 = {136} \) 。但还需计入 \( {a}^{0} \) 这一项。因此,答案为 \( {136} + 1 = (\mathbf{C}\mathbf{)}{137} \) 。
~seanyoon777
解法2
(与解法1类似)设 \( x = {2}^{17} \) 。则 \( {2}^{289} = {x}^{17} \) 。左侧可改写为 \( \frac{{x}^{17} + 1}{x + 1} = {x}^{16} - {x}^{15} + \cdots + {x}^{2} - x + 1 = \left( {x - 1}\right) \left( {{x}^{15} + {x}^{13} + \cdots + {x}^{1}}\right) + 1 \) 。将 \( {2}^{17} \) 代回
替换 \( x \) ,我们得到 \( \left( {{2}^{17} - 1}\right) \left( {{2}^{15.17} + {2}^{13.17} + \cdots + {2}^{1 \cdot {17}}}\right) + 1 = \left( {{2}^{16} + {2}^{15} + \cdots + {2}^{0}}\right) ({2}^{{15} \cdot {17}} + {2}^{{13} \cdot {17}} + \cdots + {2}^{1.1} \) 。展开后,将出现 \( {17} \cdot 8 + 1 = {137} \) 项。因此,答案为 \( \left( {\mathbf{C}}\right) {137} \)
解法3(直观)
两边同乘 \( {2}^{17} + 1 \) ,得到
\[ {2}^{289} + 1 = {2}^{{a}_{1}} + {2}^{{a}_{2}} + \ldots + {2}^{{a}_{k}} + {2}^{{a}_{1} + {17}} + {2}^{{a}_{2} + {17}} + \ldots + {2}^{{a}_{k} + {17}}. \]
注意到 \( {a}_{1} = 0 \) ,因为左侧有1。然而,右侧因 \( {2}^{{a}_{1} + {17}} \) 而多出一项 \( {2}^{18} \) 。为消去它,令 \( {a}_{2} = {18} \) 。两个 \( {2}^{18} \) 合并为 \( {2}^{19} \) ,于是令 \( {a}_{3} = {19} \) 。依此类推,直至 \( {a}_{18} = {34} \) 。此时,所有不需要的部分缩并为 \( {2}^{35} \) 。我们已拥有该项,因为我们设
\( {a}_{2} = {18} \Longrightarrow {a}_{2} + {17} = {35} \) 。此后所有项将自动缩并为 \( {2}^{52} \) 。因此令 \( {a}_{19} \) 为52。
可见,我们需一次添加 \( {17}{a}_{n} \) ,然后“等待”其自动缩并接下来的17个数,依此类推,直至 \( {2}^{289} \) 。只需在17的奇数倍与偶数倍之间添加 \( {a}_{n} \) 。289以下最大的17的偶数倍是 \( {17} \cdot {16} \) ,故总共需添加 \( {17} \cdot 8{a}_{n} \) 。然而别忘了最初设 \( {a}_{1} = 0 \) ,因此答案为 \( {17} \cdot 8 + 1 = \) (C)137
解法4
注意该表达式略小于 \( {2}^{272} \) 。显然,选项(D)与(E)无意义,因为273项的最小和为 \( {2}^{273} - 1 \) 。(A)也毫无道理。(B)与(C)仅差1,但表达式为奇数,必含 \( {2}^{0} = 1 \) ,而(C)大1,故答案为 \( \left( \mathbf{C}\right) {137} \) 。~LCZ
解法5
为简化表达式,#表示16个连续的0。
用二进制思考此题。我们有
\( {1\# 0\# 0\# 0\# 0\# 0\# 0\# 0\# 0\# 0\# 0\# 0\# 0\# 0\# 0\# 0\# 0\# 0\# 1}{1}_{2} \)
\( 1\# {1}_{2} \)
注意 \( \left( {{2}^{17} + 1}\right) \left( {{2}^{0} + {2}^{34} + {2}^{68} + \cdots + {2}^{272}}\right) = {2}^{0}\left( {{2}^{17} + 1}\right) + {2}^{34}\left( {{2}^{17} + 1}\right) + {2}^{68}\left( {{2}^{17} + 1}\right) + \cdots {2}^{272}\left( \right. \)
\( = {1\# 1\# 1\# 1\# 1\# 1\# 1\# 1\# 1\# 1\# 1\# 1\# 1\# 1\# 1\# 1\# 1\# 1\# 1\# 1\# 1} \)
\( ({2}^{17} + 1)({2}^{17} + {2}^{51} + {2}^{85} + \cdots + {2}^{255}) = {2}^{17}({2}^{17} + 1) + {2}^{51}({2}^{17} + 1) + {2}^{85}({2}^{17} + 1) + \cdots {2}^{25} \)
= 1#1#1#1#1#1#1#1#1#1#1#1#1#1#1#1#1#1#0
由于
]#1#1#1#1#1#1#1#1#1#1#1#1#1#1#1#1#1#1#1
\( - \;\begin{matrix} 1 & \# & 1 & \# & 1 & \# & 1 & \# & 1 & \# & 1 & \# & 1 & \# & 1 & \# & 1 & \# & 1 & \# & 1 & \# & 1 & \# & 1 & \# & 1 & \# & 1 & \# & 1 & \# & 1 & \# & 0 \end{matrix} \)
\( = 1\# 0\# 0\# 0\# 0\# 0\# 0\# 0\# 0\# 0\# 0\# 0\# 0\# 0\# 0\# 0\# 1\# 1 \)
这意味着
\( \left( {{2}^{17} + 1}\right) \left( {{2}^{0} + {2}^{34} + {2}^{68} + \cdots + {2}^{272}}\right) - \left( {{2}^{17} + 1}\right) \left( {{2}^{17} + {2}^{51} + {2}^{85} + \cdots + {2}^{255}}\right) = {2}^{289} \)
所以 \( \frac{{2}^{289} + 1}{{2}^{17} + 1} = ({2}^{0} + {2}^{34} + {2}^{68} + \cdots + {2}^{272}) - ({2}^{17} + {2}^{51} + {2}^{85} + \cdots + {2}^{255}) \)
\( = {2}^{0} + \left( {{2}^{34} - {2}^{17}}\right) + \left( {{2}^{68} - {2}^{51}}\right) + \cdots + \left( {{2}^{272} - {2}^{255}}\right) \)
将每对形如 \( {2}^{n + {17}} - {2}^{n} \) 的数用二进制表示,我们得到
\( {10000000000000000000}\cdots {0}_{2} \)
\( {10}\cdots {0}_{2} \)
\( = \left\lbrack \begin{matrix} 1 \\ 1 \\ 1 \\ 1 \\ 1 \\ 1 \\ 1 \\ 1 \\ 1 \\ 1 \\ 1 \\ 1 \\ 1 \\ 1 \\ 1 \\ 0 \\ \cdots \\ \end{matrix}\right\rbrack \cdot {0}_{2}^{ - } \)
或
\( {2}^{n + {17}} - {2}^{n} = {2}^{n + {16}} + {2}^{n + {15}} + {2}^{n + {14}} + \cdots + {2}^{n} \)
这意味着每一对包含17个形如 \( {2}^{n} \) 的项。
由于共有8对,因此总共有 \( 8 \cdot {17} = {136} \) 项。再计入未配对的 \( {2}^{0} \) 项,
我们共有 \( {136} + 1 = \left( {\mathbf{C}\mathbf{\sum }}\right) {137} \) 项。-emerald_block
视频解析
https://youtu.be/Ozp3k2464u4
~IceMatrix
另见
2020 AMC 10A 第22题
题目
有多少个正整数 \( n \leq {1000} \) 满足
\[ \left\lfloor \frac{998}{n}\right\rfloor + \left\lfloor \frac{999}{n}\right\rfloor + \left\lfloor \frac{1000}{n}\right\rfloor \]
不能被3整除?(回忆: \( \lfloor x\rfloor \) 表示不大于 \( x \) 的最大整数。)
(A) 22 (B) 23 (C) 24 (D) 25 (E) 26
解法1(分类讨论)
表达式:
\[ \left\lfloor \frac{998}{n}\right\rfloor + \left\lfloor \frac{999}{n}\right\rfloor + \left\lfloor \frac{1000}{n}\right\rfloor \]
解答:
令 \( a = \left\lfloor \frac{998}{n}\right\rfloor \)
由于 \( \frac{1000}{n} - \frac{998}{n} = \frac{2}{n} \) ,对于任意整数 \( n \geq 2 \) ,最大差值
在应用 \( \lfloor x\rfloor \) 函数之前,最大项与最小项之差小于或等于1,因此应用该函数后,各项的取值范围必须为1或更小。
这意味着对于每一个整数 \( n \geq 2 \) , \( \bullet \) 如果 \( \frac{998}{ - } \) 是整数且 \( n \neq 2 \) ,则上述表达式中的三项必须为 \( \left( {a, a, a}\right) , \) \( \bullet \) ;如果 \( \frac{998}{n} \) 是整数,因为 \( n = 2 \) ,则 \( \frac{1000}{n} \) 将是整数且比 \( \frac{998}{n} \) 大1;因此表达式中的三项必须为 \( \left( {a, a, a + 1}\right) \) , \( \bullet \) ;如果 \( \frac{999}{\infty } \) 是整数,则上述表达式中的三项必须为 \( \left( {a, a + 1, a + 1}\right) \) ,
- 如果 \( \frac{1000}{n} \) 是整数,则上述表达式中的三项必须为 \( \left( {a, a, a + 1}\right) \) 和 \( \bullet \) ;如果 \( \left\{ {\frac{998}{n},\frac{999}{n},\frac{1000}{n}}\right\} \) 中没有一个是整数,则上述表达式中的三项必须为(a, a, a)。
最后一条陈述成立,因为要使各项不同,区间 \( \left( {\frac{998}{n},\frac{999}{n}}\right) \) 或区间 \( \left( {\frac{999}{n},\frac{1000}{n}}\right) \) 中必须存在某个整数。然而,这意味着将该整数乘以 \( n \) 后应得到一个新的整数,且该整数严格介于998与999之间或999与1000之间,但由于不存在这样的整数,各项不可能不同,因此必须相等。
\( \bullet \) 请注意, \( n = 1 \) 不成立;要证明这一点,只需在表达式中将 \( n \) 替换为 1。这样我们得到
\[ \left\lfloor \frac{998}{1}\right\rfloor + \left\lfloor \frac{999}{1}\right\rfloor + \left\lfloor \frac{1000}{1}\right\rfloor = {998} + {999} + {1000} = {2997} = {999} \cdot 3 \]
能被3整除。
现在,我们测试上述列出的五种情况(其中 \( n \geq 2 \) )
案例1: \( n \) 整除998且 \( n \neq 2 \)
如上所述,表达式中的三项为(a, a, a),因此它们的和为 \( {3a} \) ,可被3整除。因此,第一种情况不成立。
情况2: \( n \) 整除998且 \( n = 2 \)
如上所述,在此情况下各项必须 \( \left( {a, a, a + 1}\right) \) ,这意味着总和为 \( {3a} + 1 \) ,因此该表达式不能被3整除。因此,这是1个有效的情况。
情况3: \( n \) 整除999
因为 \( n \) 整除999,所以 \( n \) 的可能取值个数与999的因数个数相同。
999= \( {3}^{3} \cdot {37}^{1} \) 。因此,999的总因数个数为 \( 4 \cdot 2 = 8 \) 。
然而,我们必须减去1,因为 \( n = 1 \) 的情况无效,如前所述。这样就剩下 \( 8 - 1 = 7 \) 种情况。情况4: \( n \) 整除1000
因为 \( n \) 整除1000,所以 \( n \) 的可能取值个数与1000的因数个数相同。
1000= \( {5}^{3} \cdot {2}^{3} \) 。因此,1000的总因数个数为 \( 4 \cdot 4 = {16} \) 。
同样,我们必须减去1,因此剩下 \( {16} - 1 = {15} \) 种情况。我们还重复计算了因数2,因为它既被算作1000的因数,又被单独列为情况2。 \( {15} - 1 = {14} \) ,所以实际上共有14种有效情况。
情况5: \( n \) 不整除 \( \{ {998},{999},{1000}\} \) 中的任何一个
与情况1类似,表达式各项的值为(a, a, a)。总和为 \( {3a} \) ,可被3整除,因此该情况无效。
现在我们已经统计了所有情况,将它们相加。
\[ 0 + 1 + 7 + {14} + 0 = {22}\text{, so the answer is}\sigma \text{.}\sigma \text{.} \]
~dragonchomper,由emerald_block补充编辑
解法2(基本上是解法1的简化版)
- 注意此解法未统计大多数情况,因此可能不适用于其他问题
注意到只需统计1000和999的因数个数,并排除1。1000有16个因数,999有8个。相加得24,但需减去2,因为1
无效。
因此,答案是 \( {24} - 2 = \left( {\text{A},{22}}\right) \)
-happykeeper,由dragonchomper补充编辑
2020 AMC 10A 第23题
以下题目同时出现在2020 AMC 12A 第20题和2020 AMC 10A
第23题,因此两道题目均重定向至此页面。
题目
设 \( T \) 为坐标平面上以 \( \left( {0,0}\right) ,\left( {4,0}\right) , \) 和 \( \left( {0,3}\right) . \) 为顶点的三角形。考虑平面的以下五种等距变换(刚性变换):绕原点逆时针旋转 \( {90}^{ \circ },{180}^{ \circ }, \) 和 \( {270}^{ \circ } \) ,关于 \( x \) 轴的反射,以及关于 \( y \) 轴的反射。在这125种由三个变换组成的序列(允许重复)中,有多少种序列能将 \( T \) 恢复到原始位置?(例如,先进行 \( {180}^{ \circ } \) 旋转,再关于 \( x \) 轴反射,再关于 \( y \) 轴反射,可将 \( T \) 恢复到原始位置;但先进行 \( {90}^{ \circ } \) 旋转,再关于 \( x \) 轴反射,再关于 \( x \) 轴反射,则不能将 \( T \) 恢复到原始位置。)
(A) 12 (B) 15 (C) 17 (D) 20 (E) 25
解答
首先,我们所能做的任何运动组合都必须对 \( T \) 进行偶数次反射。这是因为每次反射 \( T \) 都会改变其方向。一旦 \( T \) 被翻转一次,任何旋转组合都无法将其恢复原位,因为它已成为镜像;然而,再次翻转会将其恢复到原始方向。由于我们仅允许3次变换,且其中反射次数必须为偶数,因此我们要么反射 \( T0 \) 次,要么反射2次。在此情况下,我们必须使用3次旋转才能将 \( T \) 恢复到原始位置。注意,我们的旋转集合 \( \{ {90}^{ \circ },{180}^{ \circ },{270}^{ \circ }\} \) 包含除 \( {0}^{ \circ } \) 外所有 \( {90}^{ \circ } \) 的倍数。我们可以从 \( \{ {90}^{ \circ },{180}^{ \circ },{270}^{ \circ }\} \) 中任选两个旋转 \( a, b \) ,且必存在唯一一个 \( c \equiv - a - b{\;(\operatorname{mod}\;{360}^{ \circ })} \) ,使得我们可以使用这三个旋转(a, b, c),确保 \( a + b + c \equiv {0}^{ \circ }\;\left( {\operatorname{mod}\;{360}^{ \circ }}\right) \) 。这样,旋转组合 \( a, b, c \) 将产生一个完整的旋转。例如,若 \( a = b = {90}^{ \circ } \) ,则 \( c \equiv - {90}^{ \circ } - {90}^{ \circ } = - {180}^{ \circ }{\;(\operatorname{mod}\;{360}^{ \circ })} \) ,因此 \( c = {180}^{ \circ } \) ,且旋转组合 \( \left( {{90}^{ \circ },{90}^{ \circ },{180}^{ \circ }}\right) \) 产生一个完整的旋转。
唯一失败的情况是当 \( c \) 必须等于 \( {0}^{ \circ } \) 时。这种情况发生在(a, b)本身已构成一个完整旋转时,即
\( \left( {a, b}\right) = \left( {{90}^{ \circ },{270}^{ \circ }}\right) ,\left( {{180}^{ \circ },{180}^{ \circ }}\right) , \) 或 \( \left( {{270}^{ \circ },{90}^{ \circ }}\right) \) 。然而,我们只需从总数中减去这三种情况即可。从 \( \{ {90}^{ \circ },{180}^{ \circ },{270}^{ \circ }\} \) 中选择(a, b)有 \( 3 \cdot 3 = 9 \) 种可能,减去3种失败情况后,情况1剩下6种组合。
案例2:在T上的2次反射
在此情况下,我们首先排除出现两次相同反射的可能性。由于两次关于x轴的反射会将 \( T \) 映射回自身,若在这两次反射之前、之间或之后插入一次旋转,都会改变 \( T \) 的最终位置,这意味着任何包含两次关于x轴反射的组合都无法将 \( T \) 映射回自身。两次关于y轴的反射同理。
因此,我们必须进行一次关于x轴的反射、一次关于y轴的反射以及一次旋转。由于关于x轴的反射会改变y分量的符号,关于y轴的反射会改变 \( x \) 分量的符号,而 \( {180}^{ \circ } \) 旋转会同时改变两个分量的符号,这三个变换(按任意顺序)组合起来即可满足要求。因此,问题仅在于如何排列这三者,从而在情况2中给出 \( 3! = 6 \) 种组合。
综合两种情况,我们得到 \( 6 + 6 = \) (A) 12
解决方案2(改写方案1)
与之前的解法一样,请注意,由于方向性,我们必须有0次或2次反射(reflection),因为反射会改变方向,而旋转无法修正这一点。我们还知道,不能出现两次相同的反射,否则将产生净无变化,从而需要恒等旋转(identity rotation)。
假设没有反射。为简化起见,将 \( {90}^{ \circ } \) 记为1, \( {180}^{ \circ } \) 记为2, \( {270}^{ \circ } \) 记为3。我们需要从中选取3个数的组合,使其和为4或8(0和12不可能,因为最小和为3,最大和为9)。4可通过任意排列得到,8也可通过任意排列得到。此情形共有种方式。
假设存在两个反射。如前所述,它们必须不同,因此若不进行任何旋转,三角形将被映射到原点的另一侧。然而我们还剩一次旋转可用,唯一能使其回到初始位置的旋转是2,即 \( {180}^{ \circ } \) ,亦即关于原点的反射。因此,由于所有3种变换互不相同,且这些变换可任意顺序应用(因为它们可交换,可想象为象限操作),总共有6种方式。
2020 AMC 10A 试题/第24题
问题
令 \( n \) 为大于1000的最小正整数,使得
\( \gcd \left( {{63}, n + {120}}\right) = {21} \) 和 \( \gcd \left( {n + {63},{120}}\right) = {60} \) 。
\( n \) 的各位数字之和是多少?
(A) 12 (B) 15 (C) 18 (D) 21 (E) 24
解决方案 1
我们知道 \( {gcd}\left( {{63}, n + {120}}\right) = {21} \) ,因此可以写成 \( n + {120} \equiv 0{\;(\operatorname{mod}\;{21})} \) 。化简后得到 \( n \equiv 6{\;(\operatorname{mod}\;{21})} \) 。同理,可写成 \( n + {63} \equiv 0{\;(\operatorname{mod}\;{60})} \) ,或 \( n \equiv - 3{\;(\operatorname{mod}\;{60})} \) 。解这两个同余式, \( n \equiv {237}{\;(\operatorname{mod}\;{420})} \) ,根据中国剩余定理(Chinese Remainder Theorem),这是唯一解。现在,题目要求大于1000的最小正整数,我们求得最小解为 \( n = {1077} \) 。然而,我们尚未考虑 \( {gcd}\left( {{63}, n + {120}}\right) = {63} \) 或 \( {gcd}\left( {n + {63},{120}}\right) = {120} \) 的情况。 \( {1077} + {120} \equiv 0 \) ’(mod63),因此我们尝试 \( n = {1077} + {420} = {1497} \) 。 \( {1497} + {63} \equiv 0{\;(\operatorname{mod}\;{120})} \) ,于是再次给 \( n \) 加上420。结果 \( n = {1497} + {420} = {1917} \) 确实满足原条件,故答案为 \( 1 + 9 + 1 + 7 = \) (C)18 解法2(暴力枚举)
已知 \( \gcd \left( {{63}, n + {120}}\right) = {21} \) 和 \( \gcd \left( {n + {63},{120}}\right) = {60} \) 。由此可知 \( n + {120} \) 能被21整除但不能被63整除,且 \( n + {63} \) 能被60整除但不能被120整除。首先,我们找出满足 \( n \) 条件且能被21整除的最小 \( n + {120} \) ,即1134,于是 \( n = {1014} \) 。接着,我们不断加21,直到得到一个满足第二个方程的数。这个数是1917,其各位数字之和为(C) 18
-午夜
解法3(暴力但更糟糕)
假设 \( n \) 是一个四位数。那么 \( n = {abcd} \) ,其中 \( a \) 、 \( b \) 、 \( c \) 、 \( d \) 分别表示该数的各位数字(不要与 \( a * b * c * d \) 混淆)。根据题意, \( {gcd}\left( {{63}, n + {120}}\right) = {21} \) 且 \( {gcd}\left( {n + {63},{120}}\right) = {60} \) 。因此我们知道 \( d = 7 \) ( \( n \) 的个位数字)。这意味着 \( {12} + {abc} \equiv 0{\;(\operatorname{mod}\;7)} \) 且 \( 7 + {abc} \equiv 0{\;(\operatorname{mod}\;6)} \) 。接下来我们可以暴力枚举。我们只需找出所有数对(x, y),使得 \( x \) 是7的倍数且比某个6的倍数大5。关于 \( {12} + {abc} \) 的方程为 \( {42} * j + {35} = x \) ,而关于 \( 7 + {abc} \) 的方程为 \( {42} * j + {30} = y \) ,其中 \( j \) 为任意整数。我们将该值代入,直到得到使 \( n = {abc7} \) 满足原题条件的 \( {abc} \) (记住 \( {abc} > {100} \) )。经过几分钟的暴力计算,我们发现 \( {abc} = {191} \) 成立。因此 \( n = {1917} \) ,这意味着其各位数字之和为 \( \mid \left( \mathrm{C}\right) {18} \) 。
~宝蓝
2020 AMC 10A 第25题
以下题目同时出现在2020 AMC 12A第23题和2020 AMC 10A第25题,因此两题均重定向至此页面。
题目
Jason掷三颗公平的六面骰子。然后他观察点数,并选择其中任意子集(可为空集,也可为全部三颗骰子)进行重掷。重掷后,若三颗骰子朝上面点数之和恰好为7,则获胜。Jason总是采取最优策略以最大化获胜概率。求他选择恰好重掷两颗骰子的概率。
(A) \( \frac{7}{36} \) (B) \( \frac{5}{24} \) (C) \( \frac{2}{9} \) (D) \( \frac{17}{72} \) (E) \( \frac{1}{4} \)
解法1
考虑掷两颗骰子得到和为 \( s \) 的概率,其中 \( s \leq 7 \) 。满足条件的组合有 \( s - 1 \) 种,即 \( \left( {1, s - 1}\right) ,\left( {2, s - 2}\right) ,...,\left( {s - 1,1}\right) \) ,而所有可能组合共 \( {6}^{2} = {36} \) 种。因此概率为 \( \frac{s - 1}{36} \) 。
因此,若一颗骰子点数为 \( a \) 且Jason重掷另外两颗骰子,则获胜概率为 \( \frac{7 - a - 1}{36} = \frac{6 - a}{36} \) 。
为了最大化获胜概率, \( a \) 必须最小化。这意味着若Jason重掷两颗骰子,他必须选择点数最大的两颗骰子进行重掷。
因此,我们可以令 \( a \leq b \leq c \) 为三颗骰子的点数,分别记为 \( A \) 、 \( B \) 和 \( C \) 。考虑 \( a + b < 7 \) 的情况。若 \( a + b + c = 7 \) ,则无需重掷任何骰子。否则,若只重掷一颗骰子,我们可以重掷 \( C \) ,期望掷出能使三颗骰子总和为7的点数,这种情况发生的概率为 \( \frac{1}{6} \) 。若重掷两颗骰子,我们将重掷 \( B \) 和 \( C \) ,获胜概率为 \( \frac{6 - a}{36} \) ,如前所述。然而, \( \frac{1}{6} > \frac{6 - a}{36} \) ,因此当 \( a + b < 7 \) 时,重掷一颗骰子总是比重掷两颗骰子更优。现在考虑 \( a + b \geq 7 \) 的情况。此时重掷一颗骰子无法帮助我们获胜,因为三颗骰子的总和将始终大于7。若重掷两颗骰子,获胜概率仍为 \( \frac{6 - a}{36} \) 。若要计算重掷全部三颗骰子时的获胜概率,我们可以假设每颗骰子初始为1点,然后计算将剩余4点分配到三颗骰子上的方式数。根据星与棒(Stars and Bars)方法,共有 \( \left( \begin{array}{l} 6 \\ 2 \end{array}\right) = {15} \) 种分配方式。
这使得获胜的概率为 \( \frac{15}{{6}^{3}} = \frac{5}{72} \) 。
为了让掷两个骰子比掷三个骰子更有利,
\[ \frac{6 - a}{36} > \frac{5}{72} \rightarrow a \leq 3 \]
因此,当且仅当 \( a \leq 3 \) 且 \( a + b \geq 7 \) 时,重掷两颗骰子是最优策略。满足这些条件的三元组(a, b, c)及其可排列方式共有 \( \left( {3,4,4}\right) \rightarrow 3 \) 种, \( \left( {3,4,5}\right) \rightarrow 6 \) 种。
\( \left( {3,4,6}\right) \rightarrow 6 \) 种方式。 \( \left( {3,5,5}\right) \rightarrow 3 \) 种方式。 \( \left( {3,5,6}\right) \rightarrow 6 \) 种方式。
\( \left( {3,6,6}\right) \rightarrow 3 \) 种方式。 \( \left( {2,5,5}\right) \rightarrow 3 \) 种方式。 \( \left( {2,5,6}\right) \rightarrow 6 \) 种方式。
\( \left( {2,6,6}\right) \rightarrow 3 \) 种方式。 \( \left( {1,6,6}\right) \rightarrow 3 \) 种方式。
在 \( {6}^{3} = \underline{216} \) 种可能性中,有 \( 3 + 6 + 6 + 3 + 6 + 3 + 3 + 6 + 3 + 3 = {42} \) 种情况下重掷两颗骰子是最优策略,因此概率
Jason将重掷两个骰子的概率是 \( \frac{42}{216} = \) (A) \( \frac{7}{36} \)
解决方案 2
我们计算分子。如果Jason已经掷出总和为7,他将不再捡起任何骰子。我们需要假设他一开始没有7。如果Jason决定捡起所有骰子重新掷,根据星条法(Stars and Bars,或随便什么方法……),会有2条和4星(其中3星必须保证,因为每次掷骰至少为1),概率为 \( \frac{15}{216} = \frac{2.5}{36} \) 。如果Jason捡起2颗骰子并留下一颗显示 \( k \) ,他需要另外两颗骰子之和为 \( 7 - k \) ,这种情况发生的概率为
\[ \frac{6 - k}{36} \]
对于整数 \( 1 \leq k \leq 6 \) 。如果掷出的点数不是7,Jason 会恰好重掷一颗骰子,只要剩下的两颗骰子之和小于7,因为这会给他带来 \( \frac{1}{6} \) 的概率,这个概率比所有其他情况都大,除非他一开始就掷出了7。我们有
\[ \frac{1}{6} > \underline{\frac{5,4,3}{36}} > \frac{2.5}{36} > \frac{2,1,0}{36}. \]
我们统计下划线部分出现的次数作为分子,同时不破坏比它更大的概率。设 \( a \) 为保留的点数。我们知道 \( a \) 最多为3,因为若为4,Jason 会重掷所有骰子。当 \( a = 1 \) 时,有3种选择决定它出现在 \( 1\mathrm{{st}},2\mathrm{{nd}} \) 还是 \( 3\mathrm{{rd}} \) ,此时另外两颗骰子必须至少为6(否则他只会重掷1颗)。这给出 \( 3 \cdot {1}^{2} = 3 \) 种方式。类似地, \( a = 2 \) 给出 \( 3 \cdot {2}^{2} = {12} \) ,因为2可以出现在3个位置,且另外两颗骰子至少为5。 \( a = 3 \) 给出 \( 3 \cdot {3}^{2} = {27} \) 。相加得到分子为42。分母为 \( {6}^{3} = {216} \) ,因此我们有 \( \frac{42}{216} = \left( A\right) \frac{7}{36} \)
视频解析
https://youtu.be/3W4jOpCiBx8 - AwesomeMath_Life 另见